SOLUTION: Let R=(a,b) be the image of rotating point P=(7,2) clockwise by 150^\circ degrees around Q=(12,-5). What is b?
Algebra.Com
Question 1210144: Let R=(a,b) be the image of rotating point P=(7,2) clockwise by 150^\circ degrees around Q=(12,-5). What is b?
Answer by CPhill(1959) (Show Source): You can put this solution on YOUR website!
Let $P = (7, 2)$ and $Q = (12, -5)$. We want to rotate $P$ clockwise by $150^\circ$ around $Q$ to obtain point $R = (a, b)$.
First, let's find the vector $\vec{QP} = P - Q = (7-12, 2-(-5)) = (-5, 7)$.
We want to rotate this vector clockwise by $150^\circ$. This is equivalent to rotating counterclockwise by $-150^\circ$.
We can represent the vector $\vec{QP}$ as a complex number: $z = -5 + 7i$.
To rotate $z$ counterclockwise by $-150^\circ$, we multiply it by $e^{-i5\pi/6} = \cos(-150^\circ) + i\sin(-150^\circ) = \cos(210^\circ) + i\sin(210^\circ) = -\frac{\sqrt{3}}{2} - \frac{1}{2}i$.
The rotated vector is:
$$z' = z \cdot e^{-i5\pi/6} = (-5 + 7i)\left(-\frac{\sqrt{3}}{2} - \frac{1}{2}i\right) = \frac{5\sqrt{3}}{2} + \frac{5}{2}i - \frac{7i\sqrt{3}}{2} + \frac{7}{2} = \left(\frac{5\sqrt{3}}{2} + \frac{7}{2}\right) + i\left(\frac{5}{2} - \frac{7\sqrt{3}}{2}\right)$$
This corresponds to the vector $\vec{QR} = \left(\frac{5\sqrt{3}}{2} + \frac{7}{2}, \frac{5}{2} - \frac{7\sqrt{3}}{2}\right)$.
So, $R = Q + \vec{QR} = \left(12 + \frac{5\sqrt{3}}{2} + \frac{7}{2}, -5 + \frac{5}{2} - \frac{7\sqrt{3}}{2}\right) = \left(\frac{31}{2} + \frac{5\sqrt{3}}{2}, -\frac{5}{2} - \frac{7\sqrt{3}}{2}\right)$.
We want to find $b$, which is the $y$-coordinate of $R$.
$$b = -\frac{5}{2} - \frac{7\sqrt{3}}{2} = \frac{-5 - 7\sqrt{3}}{2}$$
Final Answer: The final answer is $\boxed{\frac{-5-7\sqrt{3}}{2}}$
RELATED QUESTIONS
Let R be the image of rotating point P=(4,0) counterclockwise by 60^\circ degrees around... (answered by CPhill)
what are the coordinates of the terminal point P after rotating 315 degrees clockwise... (answered by Fombitz,ewatrrr)
what are the coordinates of R, the image p(1,3) under counter clockwise rotation 90... (answered by Edwin McCravy)
Find the image of point (5, 3) when it is rotated 1/4 of a turn counter-clockwise... (answered by math_tutor2020,greenestamps)
3) show that if A-B-C and B-C-D,then A-B-D and A-C-D
4)Given aline m with coordinate... (answered by 90089)
Please help me solve this equation:
Let R be the region enclosed by {{{ y=-(x-1)^2+1 }}} (answered by solver91311)
1. If y(x-1) = z then x=
A. y-z
B. z/y + 1
C. y(z-1)
D. z(y-1)
2. If 3x=6x-15 then x (answered by Fombitz)
In right triangle ABC, AC = BC and angle C = 90. Let P and Q be points on hypotenuse AB,... (answered by foofy)
In right triangle ABC, AC = BC and ∠C = 90. Let P and Q be points on hypotenuse
AB,... (answered by josgarithmetic,Edwin McCravy,AnlytcPhil)